Γεωμετρική Ανισότητα!

Συντονιστές: achilleas, emouroukos, silouan

Άβαταρ μέλους
big-pitsirikos
Δημοσιεύσεις: 59
Εγγραφή: Τετ Οκτ 19, 2016 11:25 am

Γεωμετρική Ανισότητα!

#1

Μη αναγνωσμένη δημοσίευση από big-pitsirikos » Τετ Οκτ 19, 2016 11:03 pm

Σε τρίγωνο ABC, με ύψη h_a,h_b,h_c και ακτίνα περιγεγραμμένου κύκλου R, να δείξετε ότι :

h_a+h_b+h_c \leq \dfrac{27R}{6}.


Αλίμονο σ'αυτούς που δεν ξέρουν ότι δεν ξέρουν αυτά που δεν ξέρουν !

Λέξεις Κλειδιά:
Άβαταρ μέλους
matha
Γενικός Συντονιστής
Δημοσιεύσεις: 6423
Εγγραφή: Παρ Μάιος 21, 2010 7:40 pm
Τοποθεσία: Θεσσαλονίκη

Re: Γεωμετρική Ανισότητα!

#2

Μη αναγνωσμένη δημοσίευση από matha » Τετ Οκτ 19, 2016 11:49 pm

\displaystyle{h_a+h_b+h_c=\frac{2E}{a}+\frac{2E}{b}+\frac{2E}{c}=2E\left(\frac{1}{a}+\frac{1}{b}+\frac{1}{c}\right)=\frac{abc}{2R}\frac{ab+bc+ca}{abc}=\frac{ab+bc+ca}{2R},}

οπότε αρκεί \displaystyle{ab+bc+ca\leq 9R^2.}

Αυτή ισχύει: \displaystyle{ab+bc+ca\leq a^2+b^2+c^2\leq 9R^2,} με τη δεξία να είναι π.χ. συνέπεια της \displaystyle{OG^2=R^2-\frac{a^2+b^2+c^2}{9}\geq 0.}

\displaystyle{\rule{500pt}{1pt}}

Διαφορετικά είναι συνέπεια των εξής:

\displaystyle{\bullet} \displaystyle{h_a\leq \sqrt{s(s-a)}}

\displaystyle{\bullet} \displaystyle{\sqrt{x}+\sqrt{y}+\sqrt{z}\leq \sqrt{3(x+y+z)}}

\displaystyle{\bullet} \displaystyle{3\sqrt{3}R\geq 2s.}

\displaystyle{\rule{500pt}{1pt}}


Θεωρώ ότι δεν είναι επιπέδου διαγωνισμών γυμνασίου, τουλάχιστον για τα ελληνικά δεδομένα.


Μάγκος Θάνος
Άβαταρ μέλους
Demetres
Γενικός Συντονιστής
Δημοσιεύσεις: 8989
Εγγραφή: Δευ Ιαν 19, 2009 5:16 pm
Τοποθεσία: Λεμεσός/Πύλα
Επικοινωνία:

Re: Γεωμετρική Ανισότητα!

#3

Μη αναγνωσμένη δημοσίευση από Demetres » Πέμ Οκτ 20, 2016 12:07 am

matha έγραψε:
Θεωρώ ότι δεν είναι επιπέδου διαγωνισμών γυμνασίου, τουλάχιστον για τα ελληνικά δεδομένα.
Μεταφέρθηκε στους Seniors.

Θάνο, έχεις κάποια τυπογραφικά στην αρχή της πρώτης γραμμής και στο τέλος της τρίτης.


harrisp
Δημοσιεύσεις: 546
Εγγραφή: Σάβ Μαρ 28, 2015 8:49 pm

Re: Γεωμετρική Ανισότητα!

#4

Μη αναγνωσμένη δημοσίευση από harrisp » Πέμ Οκτ 20, 2016 2:22 pm

Mια διαφορετική προσσέγγιση για το θέμα...

Είναι γνωστό:

\dfrac {h_{a}+h_{b}+h_{c}}{3}\leq \dfrac {m_{a}+m_{b}+m_{c}}{{ 3 }}\leq \sqrt {\dfrac {m_{a}^{2}+m_{b}^{2}+m_{c}^{2}} {3}}

Αντικαθιστώντας με την βοήθεια του θεωρήματος των διαμέσων και χρήση του τύπου:

a^{2}+b^{2}+c^{2}\leq 9R^{2} έπεται το ζητούμενο.



Edit: Προστέθηκε το 3 στον παρονομαστή. Ευχαριστώ τον κύριο Σταύρο για την επισήμανση.
τελευταία επεξεργασία από harrisp σε Πέμ Οκτ 20, 2016 6:18 pm, έχει επεξεργασθεί 6 φορές συνολικά.


ΠΑΠΑΔΟΠΟΥΛΟΣ ΣΤΑΥΡΟΣ
Δημοσιεύσεις: 3600
Εγγραφή: Πέμ Φεβ 27, 2014 9:05 am
Τοποθεσία: ΧΑΛΚΙΔΑ- ΑΘΗΝΑ-ΚΡΗΤΗ

Re: Γεωμετρική Ανισότητα!

#5

Μη αναγνωσμένη δημοσίευση από ΠΑΠΑΔΟΠΟΥΛΟΣ ΣΤΑΥΡΟΣ » Πέμ Οκτ 20, 2016 2:35 pm

ΧΑΡΗΣ ΤΙΟΥΡΙΝΓΚ έγραψε:Mια διαφορετική προσσέγγιση για το θέμα...

Είναι γνωστό:

h_{a}+h_{b}+h_{c}\leq m_{a}+m_{b}+m_{c}\leq \sqrt {\dfrac {m_{a}^{2}+m_{b}^{2}+m_{c}^{2}} {3}}
Για ξανακοίτα την τελευταία ανισότητα.


Άβαταρ μέλους
george visvikis
Επιμελητής
Δημοσιεύσεις: 13298
Εγγραφή: Παρ Νοέμ 01, 2013 9:35 am

Re: Γεωμετρική Ανισότητα!

#6

Μη αναγνωσμένη δημοσίευση από george visvikis » Πέμ Οκτ 20, 2016 6:46 pm

Κάτι παρεμφερές με τον Θάνο.

\displaystyle{2E = 2rs \leqslant 3Rr\sqrt 3 } (1)

\displaystyle{\frac{1}{a} + \frac{1}{b} + \frac{1}{c} \leqslant \frac{{\sqrt 3 }}{{2r}}} (2)

\displaystyle{{h_a} + {h_b} + {h_c} = 2E\left( {\frac{1}{a} + \frac{1}{b} + \frac{1}{c}} \right)\mathop  \Leftrightarrow \limits^{(1),(2)} {h_a} + {h_b} + {h_c} \leqslant \frac{{9R}}{2}}


Απάντηση

Επιστροφή σε “Άλγεβρα - Προχωρημένο Επίπεδο (Seniors)”

Μέλη σε σύνδεση

Μέλη σε αυτήν τη Δ. Συζήτηση: Δεν υπάρχουν εγγεγραμμένα μέλη και 4 επισκέπτες